2020-2021/TST/DM/2102_DM2/TST1/corr_02_2102_DM2.tex

204 lines
7.5 KiB
TeX

\documentclass[a5paper,10pt]{article}
\usepackage{myXsim}
\usepackage{tasks}
% Title Page
\title{DM2 \hfill BAHBAH Zakaria}
\tribe{TST}
\date{\hfillÀ render pour le Mercredi 24 février}
\xsimsetup{
solution/print = true
}
\begin{document}
\maketitle
\begin{exercise}[subtitle={Loi binomiale}]
Trois personnes s'apprêtent à passer le portique de sécurité. On suppose que pour chaque personne la probabilité que le portique sonne est égale à $0.25$.
Soit $X$ la variable aléatoire donnant le nombre de personnes faisant sonner le portique, parmi les 3 personnes de ce groupe.
\begin{enumerate}
\item Tracer l'arbre représentant le situation.
\item Justifier que $X$ suit une loi binomiale dont on précisera les paramètres.
\item Quelle est la probabilité qu'une seule personne fasse sonner le portique?
\item Calculer puis interpréter les probabilités suivantes
\[
P(X = 0) \qquad \qquad P(X \geq 2)
\]
\item Calculer l'espérance de $X$ et interpréter le résultat.
\end{enumerate}
\end{exercise}
\begin{solution}
\begin{enumerate}
\item
\begin{tikzpicture}[sloped]
\node {.}
child {node {$0$}
child {node {$0$}
child {node {$0$}
edge from parent
node[above] {0.75}
}
child {node {$1$}
edge from parent
node[above] {0.25}
}
edge from parent
node[above] {0.75}
}
child[missing] {}
child {node {$1$}
child {node {$0$}
edge from parent
node[above] {0.75}
}
child {node {$1$}
edge from parent
node[above] {0.25}
}
edge from parent
node[above] {0.75}
}
edge from parent
node[above] {0.75}
}
child[missing] {}
child[missing] {}
child[missing] {}
child { node {$1$}
child {node {$0$}
child {node {$0$}
edge from parent
node[above] {0.75}
}
child {node {$1$}
edge from parent
node[above] {0.25}
}
edge from parent
node[above] {0.75}
}
child[missing] {}
child {node {$1$}
child {node {$0$}
edge from parent
node[above] {0.75}
}
child {node {$1$}
edge from parent
node[above] {0.25}
}
edge from parent
node[above] {0.75}
}
edge from parent
node[above] {0.25}
} ;
\end{tikzpicture}
\item Chaque personne a 2 possibilités (1: fait sonner ou 2: ne fait pas sonner) et l'on fait passer 3 personnes ce qui correspond à une répétition identique et aléatoire. On peut donc modéliser la situation par une loi binomiale.
\[
X \sim \mathcal{B}(3; 0.76)
\]
\item Probabilité qu'une seule personne fasse sonner le portique. On voit qu'il y a 3 branches qui correspondent à cette situation dont
\[
P(X = 1) = 3 \times 0.25^1 \times 0.75^2 \approx 0.422
\]
\item
\[
P(X = 0) = 0.75^3 \approx 0.422
\]
\[
P(X \geq 2) = P(X = 2) + P(X = 3) = 3 \times 0.25^2 \times 0.75^1 + 0.25^3 \approx 0.157
\]
\item Il faut d'abord tracer le tableau résumant la loi de probabilité:
\begin{center}
\begin{tabular}{|c|*{4}{c|}}
\hline
Valeur & 0 & 1 & 2 & 3 \\
\hline
Probabilité & $0.422$ & $0.422$ & $0.141$ &$0.016$ \\
\hline
\end{tabular}
\end{center}
On peut alors calculer l'espérance
\[
E[X] = 0 \times 0.422 + 1 \times 0.422 + 2 \times 0.141 + 3 \times 0.016 = 0.75
\]
On peut donc estimer qu'il y aura en moyenne $0.75$ personnes qui feront sonner le portique sur les 3 personnes.
\end{enumerate}
\end{solution}
\begin{exercise}[subtitle={Équation puissance}]
Résoudre les équations et inéquations suivantes
\begin{multicols}{2}
\begin{enumerate}
\item $10^x = 6$
\item $4^x = 46$
\item $0.74^x \leq 39$
\item $4 \times 0.52^x = 19$
\end{enumerate}
\end{multicols}
\end{exercise}
\begin{solution}
Les solutions ci-dessous ne sont pas justifiée car l'ordinateur ne sait pas faire. Par contre, vous vous devez savoir justifier vos réponses!
\begin{enumerate}
\item $x = \log(6)$
\item $x = \frac{\log(46)}{\log(4)}$
\item Il faut faire attention quand on divise par un log car ce dernier peut être négatif ce qui est le cas ici. Il faut donc pense à changer le sens de l'inégalité.
$x \geq \frac{\log(39)}{\log(0.74)}$
\item Il faut penser à faire la division à par $4$ avant d'utiliser le log car sinon, on ne peut pas utiliser la formule $\log(a^n) = n\times \log(a)$.
$x = \frac{\log(4.75)}{\log(0.52)}$
\end{enumerate}
\end{solution}
\begin{exercise}[subtitle={Étude de fonctions}]
Soit $f(x) = 10x^3 - 600x^2 - 15000x + 35$ une fonction définie sur $\R$.
\begin{enumerate}
\item Calculer $f'(x)$ la dérivée de $f(x)$.
\item Calculer $f'(50)$ et $f'(-10)$.
\item En déduire une forme factorisée de $f'(x)$.
\item Étudier le signe de $f'(x)$ et en déduire les variations de $f(x)$.
\item Est-ce que la fonction $f(x)$ admet un maximum ou un minimum? Si oui, calculer sa valeur.
\end{enumerate}
\end{exercise}
\begin{solution}
\begin{enumerate}
\item Dérivée de $f(x)$: $f'(x) = 30x^2 - 1200x - 15000$
\item
\begin{align*}
f'(50) &= 30 \times 50^{2} - 1200 \times 50 - 15000\\&= 30 \times 2500 - 60000 - 15000\\&= 75000 - 75000\\&= 0
\end{align*}
\begin{align*}
f'(-10) &= 30 \times - 10^{2} - 1200(- 10) - 15000\\&= 30 \times 100 + 12000 - 15000\\&= 3000 - 3000\\&= 0
\end{align*}
Donc $x = 50$ et $x=-10$ sont des racines de $f'(x) = 30x^2 - 1200x - 15000$.
\item On en déduit la forme factorisée suivante
\[
f'(x) = 30 (x - 50)(x--10)
\]
\item Pas de correction disponible
\item À causes des branches extérieurs, la fonction $f(x)$ n'a pas de maximum ou de minimum.
\end{enumerate}
\end{solution}
%\printsolutionstype{exercise}
\end{document}
%%% Local Variables:
%%% mode: latex
%%% TeX-master: "master"
%%% End: